Limit of a Sequence 01

Calculus Level pending

Let a n a_n be a real-valued sequence such that 1 < a n < 1 \displaystyle{-1 < a_n < 1} for all n N n \geq N , for some N N N \in \mathbb{N} . Which of the following is the STRONGEST true statement that can be made about a n a_n ?

None of these are necessarily true. lim n a n \lim_{n \rightarrow \infty} a_n exists and 1 < lim n a n < 1 -1 < \lim_{n \rightarrow \infty} a_n< 1 lim n a n \lim_{n \rightarrow \infty} a_n exists and 1 lim n a n 1 -1 \leq \lim_{n \rightarrow \infty} a_n\leq 1 lim n a n \lim_{n \rightarrow \infty} a_n exists and is finite.

This section requires Javascript.
You are seeing this because something didn't load right. We suggest you, (a) try refreshing the page, (b) enabling javascript if it is disabled on your browser and, finally, (c) loading the non-javascript version of this page . We're sorry about the hassle.

1 solution

Daniel Juncos
Sep 17, 2017

Let a n = ( 1 ) n 2 \displaystyle{a_n = \frac{(-1)^n}{2}} . The a n a_n is 1 2 , 1 2 , 1 2 , 1 2 , . . . \displaystyle{\frac{1}{2}, \frac{-1}{2}, \frac{1}{2}, \frac{-1}{2}, ...} ; i.e. 1 < a n < 1 -1 < a_n < 1 for all n 0 n \geq 0 , but lim n a n \displaystyle{\lim_{n \rightarrow \infty} a_n} does not exist.

0 pending reports

×

Problem Loading...

Note Loading...

Set Loading...